[Physics] Infinite series of derivatives of position when starting from rest

calculuskinematicsmathematical physicsmathematics

Suppose you have an object with zero for the value of all the derivatives of position. In order to get the object moving you would need to increase the value of the velocity from zero to some finite value. A change is velocity is acceleration, so the value of the acceleration would have to increase from zero to some value. You would also need to increase the jerk from zero to some value to have a change in acceleration. Is there an infinite series of higher derivatives of position for this to work? Or am I missing something?

Best Answer

I) Disclaimer: In this answer, we will only consider the mathematically idealized classical problem, which is interesting in itself, although admittedly academic and detached from actual physical systems.

II) It is in principle possible that all time derivatives of the position $x(t)$ vanishes at $t=0$, and yet the particle does move away (from where it was at $t=0$).

Imgur

$\uparrow$ Fig. 1: A plot of position $x$ as a function of time $t$.

Example: Assume that the position is given by the following infinitely many times differentiable function

$$ x(t)~:=~\left\{ \begin{array}{ccl} \exp\left(-\frac{1}{|t|}\right)&\text{for} & t~\neq~ 0, \\ \\ 0 &\text{for} & t~=~ 0. \end{array} \right. $$

Note that the Taylor series for the $C^{\infty}$-function $x:\mathbb{R}\to\mathbb{R}$ in the point $t=0$ is identically equal to zero.$^1$ So the function $x$ and its Taylor series are different for $t\neq 0$. In particular, we conclude that the smooth function $x$ is not a real analytic function.

III) If you like this Phys.SE question you may also enjoy reading about What situations in classical physics are non-deterministic? , Non-uniqueness of solutions in Newtonian mechanics and Norton's dome and its equation.

--

$^1$ Sketched proof that $x\in C^{\infty}(\mathbb{R})$: Firstly, obviously, $x$ is $C^{\infty}$ for $t\neq 0$. By induction in $n\in\mathbb{N}_0$, for $t\neq 0$, it is straightforward to deduce that the $n$'th derivative is of the form $$x^{(n)}(t)~=~\frac{P_n(t,|t|)}{Q_n(t,|t|)}x(t), \qquad t\neq 0,$$ for some polynomials $P_n$ and $Q_n\neq 0$. Secondly, it follows that the $(n\!+\!1)$'th derivative at the origin $$x^{(n+1)}(0)~=~\lim_{t\to 0} \frac{x^{(n)}(t)}{t}~=~0$$ exists and is zero "because exponentials beat polynomials". $\Box$

Related Question